what is 2x + 27 = -3 (x+5) + 7​

Answers

Answer 1

Answer:

The answer is x = -7

Answer 2

Answer:

2^x = 16

Step-by-step explanation:


Related Questions

It takes 3 2 ⁄6 spoons of chocolate syrup to make 2 3 ⁄5 gallons of chocolate milk. How many spoons of syrup would it take to make 4 gallons of chocolate milk?

Answers

Answer:

Step-by-step explanation:

This is  a proportion question.

change 2.6 to a decimal. 2/6 = 1/3 = 0.3333333

change 3/5 to a decimal 3/5 = 0.6

3.3333 / 2.6 = x / 4                 Cross multiply

4*3.33333 = 2.6x                    Combine the left.

13.33333= 2.6x                       Divide by 2.6

13.3333/2.6 = x

x = 5.128 spoons of syrup.

can somebody help me with this please.​

Answers

I don't know what number 1 and number 2 mean,

3:11

4

5:-3

6:-8

You can find the answer below. Among the questions I answered correctly, I hope to adopt it at the end. Thank you. Have a nice day.

Please help I need help fast

Answers

Answer:This problem asks how many groups of 1/3 are in 4.

Quotient :12

Step-by-step explanation:

Prove that ABCD is a rectangle.
A = (-4, 4)
B = (0,5)
C = (1, 1)
D = (-3,0)

Answers

Answer:

Draw it on a graph sheet

The easiest way to prove it is to show it on a graph paper

how do you turn 0.03 into a fraction if the 3 is repeating​

Answers

Answer:

it is 1/30

Step-by-step explanation:

Lets say x =0.03 repeating

100x=3.3333....

-  10x=0.333....

_____________

90x=3

Then divide both sides by 9 to get x:

x=3/90

simplify:

x=1/30

ARIANA Y SU FAMILIA HAN CONCRETADO UN PROYECTO FAMILIAR DENOMINADO INTALACION DE UNA GRANJA RN LA QUE CREIAN CORDONICES . ELLOS VENDEN LOS HUEVOS EN CAJA DE 20 UNIDADES . SI LA ULTIMA VEZ VENDIERON 27CAJAS ¿CUANTOS VENDIERON EN TOTAL?

Answers

Answer:

540 unidades

Step-by-step explanation:

De la pregunta,

Los huevos se venden:

1 caja = 20 unidades

La última vez que vendiste 27 cajas. Por lo tanto,

1 caja = 20 unidades

27 cajas = x unidades

Cruz multiplicar

x = 27 × 20 unidades

x = 540 unidades.

Por lo tanto, se vendieron 540 unidades de huevos en total.

What is the y-intercept and x-intercept of 2x + y = 10x - 1

Answers

Answer:

Wait I only see the part where it says “what is the y-intercept and x-intercept of” but I don’t see an equation :/

Step-by-step explanation:

To begin a basketball game, a referee msut toss the ball vertically into the air. This process can be modeled by the equation h(t) = -16t^2+22t+6, where h represents the ball's height (in feet) after t seconds. Determine the time interval (in seconds) in which the height of the basketball is 8 feet. Round your answer to the nearest thousandth of a second

Answers

Answer:

1.277s and 0.098s

Step-by-step explanation:

Given the height of the ball modelled by the equation

h(t) = -16t^2+22t+6, where;

t is the time in seconds

To calculate the time interval (in seconds) in which the height of the basketball is 8 feet, we will simply substitute h(t) = 8 into the expression and calculate t as shown;

8 = -16t^2+22t+6

Divide through by 2

4 = -8t^2+11t+3

-8t^2+11t+3-4 = 0

8t^2-11t+1 = 0

t = 11±√11²-4(8)/2(8)

t = 11±√121-32/16

t = 11±√89/16

t = 11±9.43/16

t = 11+9.43/16 and t = 11-9.43/16

t = 20.43/16 and t = 1.57/16

t = 1.277s and t = 0.098s

Hence the time interval to the nearest thousands are 1.277s and 0.098s

Find the gradient of the line segment between the points (-4,-5) and (2,-4).
Give your answer in its simplest form.

Answers

Answer:

1/6

Step-by-step explanation:

gradient= change in y/ change in x

-4--5/2--4= 1/6

1/6 is the gradient

The gradient of the line segment in its simplest form is: 1/6.

What is the Gradient of a Line Segment?Gradient of a line segment = slope.Slope (m) = change in y / change in x.

Given the following points, (-4,-5) and (2,-4) are on a line segment:

Gradient (m) = change in y / change in x

Gradient (m) = (-4 -(-5))/(2 -(-4))

Gradient (m) = 1/6

Therefore, the gradient of the line segment in its simplest form is: 1/6.

Learn more about gradient of a line segment on:

https://brainly.com/question/1413251

please help me in this question​

Answers

Answer:

30

Step-by-step explanation:

use Pythagoras theorem at first to find the third side of the triangle it will come 5

then use the formula to find the area of triangle

A=(5x12)/2=30cm2

Solve.

25 ‒ (‒ 42) =

A.
67
B.
17
C.
‒17
D.
‒67

Answers

A because when subtracting with a negative the subtracting sign and the negative sign cancel out causing the subtract sign and negative number to switch to positive so after that you just add the two numbers 25+42 which is 67

Answer:

You are Correct!!!

Step-by-step explanation:

what are the domain and the range ​

Answers

Answer:

Domain 1,3,5,7,9,11

Range 5,6,7,8,9,10

what is 1.54x10^21 in scientific notation

Answers

Answer:

Scientific notation: 1.54 x 10^21

Scientific e notation: 1.54e21

Real number: 1540000000000000000000

Q4. Find the range, standard deviation, and variance for the following sample data:
33, 55, 29, 40, 8, 90, 61, 41, 17, 80, 56, 17, 59, 21,78
plz answer this!

Answers

Answer:

40 I think but no I think

Joan wants to start an IRA that will have $860,000 in it when she retires in 29-years. How much should she invest annually in her IRA to do this if the interest is 8% compounded annually?

Answers

Answer:

Annual deposit= $8,271.94

Step-by-step explanation:

Giving the following information:

Future Value (FV)= $860,000

Number of periods= 29 years

Interest rate= 8% = 0.08

To calculate the annual deposit, we need to use the following formula:

FV= {A*[(1+i)^n-1]}/i

A= annual deposit

Isolating A:

A= (FV*i)/{[(1+i)^n]-1}

A= (860,000*0.08) / [(1.08^29) - 1]

A= $8,271.94

tony is making gift bags for his friends. he has 6 bags and places 9 gifts into each bag .how many gifts did he place in the bags

Answers

Answer:

6 x 9 is 54

Step-by-step explanation:

54 bags because, it says each so your multiplying.

What is being done to the variable in the equation 3 + g = -9? The number 3 is being subtracted from it. The number 3 is being added to it. The number -9 is being subtracted from it. The number -9 is being added to it.

Answers

Answer:

the number 3 is being subtracted from it because you want to get by itself

Rotate AXYZ 270° about the origin, (0,0).

Answers

Answer:

The rule for a rotation by 270° about the origin is (x,y)→(y,−x) .

A plumbing company charges $35 per hour plus a $25 travel charge for a service call. Another plumbing company charges $40 per hour for a service call with no travel charge. Solve the equation 35h+25=40h to find how many hours h a service call must be for the two companies to charge the same amount.



__ hour(s)

Answers

Answer:

5 hours

Step-by-step explanation:

35h + 25=40h

implies 25=40h-35h

25=5h

dividing both sides by 5,

h=5

If 10 cm on a map represents 25 km of actual ground, how many centimetres would 45 km of actual ground be on a map?

Answers

Answer:

The answer would be 20cm.

Factoriza la siguiente expresión 25
x2−4

Answers

Answer:

english pls

Step-by-step explanation:

La respuesta es (5x+2)(5x-2) creo
Como hacer lo;
Mira para factorizar esta problema usas;
a^2-b^2=(a+b)(a-b) adonde a=5x y luego b=2
(5x+2)(5x-2)

También disculpa si mi español está mal, hablo más el inglés que el español

Joseph deposits $950 every month into an account earning a monthly interest rate of 0.5%. How much would he have in the account after 19 months, to the nearest dollar?

Answers

Answer:

I think, 18,058 dollars.

Step-by-step explanation:

When making pasta dishes, 10 pounds of pasta makes 60 meals. How many pounds of pasta are needed to make 750 meals?

Answers

10:60=x:750

10*750=60*x
7500=60x
x=125

Write the equation that models the linear relationship

Answers

Answer:

is it degrees

Step-by-step explanation:

?

The four Western provinces cover about 2/7 of Canada's area. Alberta covers about 1/14 of Canada's area. What fraction describes how much of Canada's area is covered by British Columbia, Manitoba, and Saskatchewan? Explain

Answers

Answer:5/14

Step-by-step explanation:if you're just combining them all 2/7 is equal to 4/14 and add that to the existing 1/14 so 4/14+1/14=5/14 total area covered

What is x and y ?

I’ll give brainliest!

The options are on the side of the screen/triangle..

Answers

Answer:

I'm pretty sure x is 3

Step-by-step explanation:

what is simplified expression for the expression below? 4(x+8)+5(x-3)

A) 9x+5
B) 9x+11
C)9x+17
D) 9x+47

Answers

4(x+8)+5(x-3)

4x+32+5x-15

9x+17

The answer is CCCCCCCC

[tex]28-(3x+4)=2(x+6)+x[/tex]

Answers

Answer:

-47

Step-by-step explanation:

Please Help!!!!!!!!!!
Is each line parallel, perpendicular, or neither parallel nor perpendicular to a line whose slope is 2/5?
line n, with slope -5/2
line m, with slope 5/2
line p, with slope 2/5

Answers

Answer:

Step-by-step explanation:

n is perpendicular. - 1/(2/5) = - 5/2. n is the negative reciprocal of the given line.

m is neither.

p is parallel to the given line. For example y = 2/5x

and y = 2/5 x + 9  are parallel to each other.

What would this be? PS it's multiple choice.

Answers

Answer:

I think it would be A. the first one

Step-by-step explanation:

tell me if it was correct if it wasnt im sorry hope i dont mess u up

Other Questions
A annoying small child is dropped off a 16.0 m cliff (for complaining about homework). Thespeed the child will reach right before they hit the ground ism/s - Don't worry,the child will not sustain life threatening injuries as they ended up doing all their homeworkand therefore was saved by the magical Physics Creature.(Record your answer to correct significant digits). have anybody done this ? Your school has been awarded a grant and plans to use the money to purchase smart devices for students. Write an essay in which you argue whether or not this technology purchase will improve students lives. Your essay must be based on ideas and information that can be found in the Do Smart Devices Improve Student Lives? passage set. if yes can you help me out i been missing school because for so many doctor appointments i had ........ Two identical bars are conducting heat from a region of higher temperature to one of lower temperature. In arrangement A, the bars conduct 80 J of heat in a certain amount of time. How much heat is conducted in B during the same time Susie cooked sausages on a barbecue. Fat and water in the sausages changed state. which two changes of state occur during this process? a. Fat melted b. Water evaporated What helped make farming easier? Helppp A tailor charge $560 for a suit of clothes and gives a discount of 12% for cash. Calculate the discounted price for a suit of clothes. Which of the following refers to the elapsed time to respond to stimulus, such as responding while driving a car or catching a ball?A. AgilityB. PowerC. CoordinationD. Reaction time A gram of gasoline produces 45 kJ of energy when burned. Gasoline has a density of 0.77 g/mL. How would you calculate the amount of energy produced by burning 48 L of gasoline? As global trade expanded in the 16th and 17th centuries, which of the following economies exported more than it imported, thus absorbing a large amount of the worlds silver?ChinaChinaPortugalPortugalJapanJapanSpain PLZ HURRY IT'S URGENT!!Which equation matches the scenario?Maggie has $15 to spend. Pinball games cost $0.75 per play and skee-ball games cost $1.25 per play. Question 4 options:A. 15x+0.75y1.25B. 0.75x+1.25y15C. (0.75+1.25)x i need help plssss thank uu A recipe states that 5 servings of peanuts have 1,250 calories. How many servings is 375 calories of peanuts? * Answer PLEASE ASAPquestion on the image. State one misconception the author presents about reading science fiction. How does the author refute the misconception? Use at least two examples from the text to support your response. Select all angles of symmetry. Given the following information, write the linear equation in slope-intercept form. 3. slope = 1/3; y-intercept = -14. slope = 2/5; y-intercept = 0 5. slope = -1; y-intercept = 4 Which of the following was NOT a factor that led to the Mexican armys loss at San Jacinto? a. The Mexican army was trapped from escape by rivers and swamp land. b. The Mexican army was caught off guard during siesta. c. The Mexican army was outnumbered by Texan forces. d. The Mexican army did not see the Texans approaching from behind the tree coverage. please help me please please Which number is closest to 22 ? .How does the pattern of light that results when it passes through the slits in the double-slit experiment support the wave model of light?A. .It is the result of refraction.B.It is the result of diffraction.C.It is the result of the photoelectric effect.D.It is the result of blackbody radiation.